LSAT and Law School Admissions Forum

Get expert LSAT preparation and law school admissions advice from PowerScore Test Preparation.

 Administrator
PowerScore Staff
  • PowerScore Staff
  • Posts: 8949
  • Joined: Feb 02, 2011
|
#22856
Complete Question Explanation

Must Be True. The correct answer choice is (A)

In the U.S. , joint safety committees are much less common than they are in Canada and Sweden , where they are required by law for all medium-sized and large companies. The U.S. also ranks far behind these countries when it comes to workplace safety. Given that joint safety committees have a proven record of reducing occupational injuries in all three countries, it is reasonable to infer that increasing their number in medium-sized and large workplaces in the U.S. would further benefit workplace safety in that country. Answer choice (A) is therefore correct.

In fact-based stimuli such as this one, it is imperative to put the facts together and prephrase a logically valid conclusion before looking at the answer choices.

Answer choice (A): This is the correct answer choice. See discussion above.

Answer choice (B): It is unclear whether it is the mandatory nature of joint safety committees in Sweden and Canada that reduces occupational injuries in these countries or whether it is simply their ubiquitous nature. Given that the committees have had a positive impact on workplace safety in all three countries (including the U.S. , where they are voluntarily established), it is reasonable to suspect that their effectiveness has a lot more to do with the sheer number of companies that have them. This answer choice is incorrect.

Answer choice (C): This answer choice would be a good way to weaken the argument, as it suggests that workplace safety may have little to do with the establishment of joint safety committees in medium-sized and large workplaces. However, since we are required to assume that the information in the stimulus is true, this answer choice is illogical and incorrect.

Answer choice (D): The stimulus provides no evidence as to whether the committees had been voluntarily established in Sweden and Canada prior to the passage of laws requiring such committees. This answer choice is irrelevant and incorrect.

Answer choice (E): This answer choice is an exaggeration. Even though we have sufficient reason to believe that the U.S. would improve its workplace safety by requiring joint safety committees, the stimulus contains no evidence to allow us to conclude that the U.S. will surpass other countries by enacting these laws. This answer choice is incorrect.
 reop6780
  • Posts: 265
  • Joined: Jul 27, 2013
|
#15833
The correct answer is A while I chose B.

I kept both of them as contenders and I really could not decide which one was correct.

I excluded answer A because it established a causal relationship, and I felt that would be too strong.

How is it certain to conclude causal relationship like A?
- The stimuli merely presents difference between US and other European countries.

I thought answer B is not too strong and mentions the difference in stimuli.

How come is B wrong ?
 BethRibet
PowerScore Staff
  • PowerScore Staff
  • Posts: 200
  • Joined: Oct 17, 2012
|
#15837
Hi Reop,

Thanks for the question. There's actually no support in the stimulus for answer choice B. The premises indicate that use of such committees is mandated in Sweden and Canada (and not mandated in the U.S.), and that where such committees are present they create the positive effect (reduced injuries). But there's no indication that voluntary as opposed to mandatory committees are more or less effective. Just that where present, the committees work.

This of course also supports answer choice A -- the second sentence in the stimulus that such committees have been very successful in all three countries including the U.S. From that, it follows that if the committees proliferate further in U.S. workplaces, they will continue to reduce injuries, which is the gist of answer choice A.

Hope this helps!

Beth
 reop6780
  • Posts: 265
  • Joined: Jul 27, 2013
|
#15864
Hi, Beth

My questions is basically how only answer A is correct while B is incorrect even though they both seem to derive from the difference between US and other European countries.

You replied that there is no support for answer B at all. However, from line 6 the stimuli states, "in the United States, such committees are found only ...voluntarily established them. However, in Sweden...committees are required by law and exist in all medium-size and large workplaces," you see answer B does not entirely fail to address the stimuli.

Even though you mentioned the second sentence as the evidence to choose the correct answer A, I really could not see the direct support there.

The last two sentences that I quoted mention the difference between the US and other countries mentioning "medium-size and large workplaces" and "voluntarily established" opposed to "required by law."

I personally thought both A and B went a little far. I can see that the comparison made in answer B is not supported. However, answer A also make a causal relationship that is not necessarily supported - maybe it is implied in the second sentence you referred to me but I need a further explanation to understand it.

My last guess is the word "would" in answer A softens the language somehow; hence answer A does not really establish a causal relationship...?
User avatar
 KelseyWoods
PowerScore Staff
  • PowerScore Staff
  • Posts: 1079
  • Joined: Jun 26, 2013
|
#15886
Hi reop6780!

As Beth said, there really is no support for answer choice (B) because the stimulus doesn't distinguish between the effectiveness of voluntary vs. legally-required joint safety committees. Rather, the stimulus points out that because they are voluntary, only a few companies in the U.S.have joint safety committees but because they are legally-required, all companies of medium and large size in Sweden and several Canadian provinces have joint safety committees.

The second sentence tells us that joint safety committees are successful in reducing occupational injuries. If that is true, then having more joint safety committees would result in a reduction of occupational injuries, as answer choice (A) states.

Remember to accept the information in the stimulus as true in MBT questions. Because the second sentence sets up the causal relationship for us (joint safety committees cause reduction in occupational injuries), we can prove the causal relationship in answer choice (A) using the stimulus.

Hope this helps!

Best,
Kelsey
 avengingangel
  • Posts: 275
  • Joined: Jun 14, 2016
|
#29982
I ended up choosing the correct answer, but not after I had already gone down the entire list of answer choices and marked them all as Losers (including A). I did that because, I felt like A is NOT a reasonable thing to infer to be true from the facts in the stimulus... just because something works in one country does not mean it will work in another (also, that's a common correct answer choice FLAW in many LSAT questions; the false analogy!). So then, when I went back and re-read the answer choice, I chose it because I figured it would be a typical conclusion/argument made by a Speaker in LSAT questions (meaning, possibly flaws). So, that brings me to my 2nd question: when faced with a scenario like this (presented with a fact set and asked to pick the most logical conclusion), are we to choose what ACTUALLY would follow logically (based on the fact set, without any glaring flaws), or, what would follow logically from the perspective of the person making the argument (utilizing the--possibly incorrect--assumptions inherently contained in the premise). I ask because I know that, generally, we are to believe the premises to always be true, but that conclusions often can contain faulty logic.

Hope that makes sense! I'm especially concerned because the explanation to this question says "it is imperative to put the facts together and prephrase a logically valid conclusion before looking at the answer choices," and I specifically wrote out next to this question in my course book -- "WHAT'S THE PREPHRASE HERE??" :ras:

Thanks!!!

(For my own future reference: 3-120, #22)
 Adam Tyson
PowerScore Staff
  • PowerScore Staff
  • Posts: 5400
  • Joined: Apr 14, 2011
|
#30290
Good question, Angel, and an important distinction. Our goal in a question like this is to draw our own logical conclusion based on the evidence presented, whether or not we think the author would make that same conclusion. In other words, don't try to mimic the flawed logic that we typically see in LSAT stimuli, but use good logic to make an inference that is well supported by the premises (which, as you said, we accept as true on their face).

I think you'll find that answer A has more support than you at first realized. This isn't just about "what works in Canada and Sweden will work here", because we also know that it already DOES work here. The stimulus told us that in all three countries, joint safety committees have been very successful. That includes in the U.S., where they are strictly voluntary. The evidence is that there is an unwavering positive correlation between such committees and improved safety.

While that may not prove that establishing such committees in all medium and large companies in the U.S. will have results that continue to track past results, we don't need to prove it. While we lump questions like this into the broad "Must Be True" category, they actually fall into a subcategory with lower standards than that - Most Strongly Supported. In other words, it doesn't have to work for certain, it just has to be the most likely answer of the bunch if we were to base it solely on the evidence at hand.

Looked at another way, and reading the stem closely, answer A is the only one with ANY support. Even weak support is better than none, right? We have to pick A because it at least seems to get some support, despite the flaws you noted, while the other answer get none at all. Since we want the best answer (as opposed to a good answer), and since the stem said to pick the one with the most support (as opposed to the one that must be true), A is head and shoulders above the rest and must be the credited response.

By the way, as to the prephrase, how about "these committees work"?

Hope that helps!
 avengingangel
  • Posts: 275
  • Joined: Jun 14, 2016
|
#30317
Adam -- thank you !!! that was so incredibly helpful and I'm glad you understood my question perfectly. And Thanks for reiterating that sometimes we are picking most strongly supported answer, and not one that we necessarily need to prove to be true. I definitely thought the same thing about answer A (when going back the 2nd time), that it was the only answer choice with ANY support. Thanks again! This is a great lesson for me, and reminds me not to over-complicate things! (pre-phrases included)
 silent7706
  • Posts: 42
  • Joined: Mar 26, 2019
|
#64317
Hello,

I was thrown off by the word "would" in (A). In here and in LSAT world generally, does the word “would” mean definitely or possibly?

Thanks in advance.
 Adam Tyson
PowerScore Staff
  • PowerScore Staff
  • Posts: 5400
  • Joined: Apr 14, 2011
|
#64336
It depends a little on context, silent7706, but for the most part "would" all by itself (as opposed to "would likely" or "some people believe it would") is definite, certain, and absolute. If an author concludes that doing X would result in Y, then that author is completely certain that Y will occur if X occurs. It's guaranteed, in their mind at least.

Note, though, the discussion earlier in this thread about the different standards for a true Must Be True question and a softer Most Strongly Supported question like this one. Can we be absolutely certain that such a change will have the same effect in the U.S.? No. Does it seem likely, based on what we read? Is it a reasonable inference under the circumstances, supported by the information that we have? Yes. That's good enough for this question type, and so A passes that test.

Get the most out of your LSAT Prep Plus subscription.

Analyze and track your performance with our Testing and Analytics Package.